Tara looks up hotel room prices for a holiday.
A hotel has a 25% off sale on its prices.
A VAT charge of 20% must be added on at the end of the transaction after any discount.
If the regular price of a room is £80 per night, how much will Tara pay for 7 nights?

Answers

Answer 1
£560 a night for 7 days
Answer 2
Regular price £80
Discount of 25%
Price is 25% of £80 which is £60
VAT is added to it which is 20% so 20% of £60 is £72
She wants the room for 7 days so 72 *7 £144

Related Questions

The sum of three consecutive odd integers is 459. What are the integers? (show your work please)

Answers

Answer:

151+153+155

Step-by-step explanation:

First you divide 459/3.  That equals 153.  Now you need 3 integers that add to 459.  The first integer is 153 and now you subtract and add equal parts from it to get your other two.  You subtract 153 by two and get 151.  Then you add 153 by 2 and get 155.  

The perimeter of a square is 4/5 what is its area

Answers

Answer:

1/25

Step-by-step explanation:

square has 4 sides, if you add them all up you get 4/5.

1/5+1/5+1/5+1/5=4/5

so each side is 1/5

Area=length * width

A=1/5*1/5=1/25

According to the synthetic division below, which of the following statements are true? Check all that apply.
-6)2 9 -12 2 -3 11

A. When x = -6,2x2 + 9x-7 = 11.
B. When x = 6,2x2 + 9x-7 = 11.
C. (x-6) is a factor of 2x + 9x- 7.
D. When (2x + 9x-7) is divided by (x - 6), the remainder is 11.
E. When (2x2 + 9x - 7) is divided by (x + 6), the remainder is 11. F. (x+6) is a factor of 2x2 + 9x-7.​

Answers

The last number on a synthetic division represents the remainder of the division.

The true statements are:

(a). When x = -6, 2x^2 + 9x - 7 = 11. (e). When (2x2 + 9x - 7) is divided by (x + 6), the remainder is 11.

From the question, we have the dividend to b 2x^2 + 9x - 7, the last number to be 11, and the zero of the divisor to be -6.

This means that:

[tex]x = -6[/tex] --- the divisor

Set to 0

[tex]x+6 = 0[/tex]

So, when the dividend 2x^2 + 9x - 7 is divided by divisor x + 6, the remainder is 11.

This also means that, when -6 is substituted for x in the dividend, the remainder is 11

Hence, the true statements are (a) and (e)

Read more about synthetic division at:

https://brainly.com/question/12951962

someone please answer question 4 and 5!! I have 14 minutes left till the exam end automatically

Answers

4.

Suppose that the geometric mean between 6 and 12 is x thus we have :

x^2 = 6 × 12

x^2 = 72

x = + √72 Or x = - √72 ( unacceptable )

x = + √9 × √4 × √2

x = 6√2

x = 6root2

________________________________

5. similarity ratio = 24/20 = 30/25 = 6/5

Determine whether the graphs of the given equations are​ parallel, perpendicular, or neither.
y=−8
x=4
Choose the correct answer below.
A.
Perpendicular
B.
Neither
C.
Parallel

Answers

Answer:

A. Perpendicular

Step-by-step explanation:

**I recommend going to desmos; desmos is a graphing calculator and is super useful!

If lighting strikes earth 1000 times in 10 seconds how much time does it take after 7250 strikes?

Answers

to find the scale factor
7250/1000=7.25
multiple the time with the same scale factor
10 x 7.25 = 72.5 seconds

Jennifer wants to end Summer with a Water Balloon Party! She still has 26 balloons left over from last year. She wants to make sure she has more than 300 water balloons for the party this year. When she went to the store, she sees that the water balloons come in packs of 40. How many packs of water balloons does she need to buy for her party? Let b represent the number of packs of water balloons that Jennifer needs to buy. Which inequality describes the scenario?

Answers

Answer:

250 balloons will be the needed ballons

Which could be used to solve this equation?
33+n=9
Subtract 3 from both sides of the equation.
35-33+.9+33
Add 3 to both sides of the equation.
O 9+34-124

Answers

Answer:

subtract 3 1/5 from both sides of the equation

Step-by-step explanation:

Answer:

A) Subtract 3 1/5 from both sides of the equation

Step-by-step explanation:

find the slope of (1,5) (-1,-3)

Answers

Answer:

4

Step-by-step explanation:

Slope= y2-y1/x2-x1

(1,5) is placed as (x1,y1) and (-1,-3) is placed as (x2,y2)

Plug it in

-3-5/-1-1

Simplify

-8/-2 = 4

Slope=4

please help and explain these two problesm

-2a + 4 (a - b ) + 5b

x + 3 + 5 (x - 3)

Answers

Just use the distributing property and combine like terms. There isn’t a definite answer since it is an expression.

The city of Anville is currently home to 15000 people, and the population has been growing at a continuous rate of 7% per year. The city of Brinker is currently home to 13000 people, and the population has been growing at a continuous rate of 8% per year. In how many years will the populations of the two towns be equal

Answers

10400 people

X2 =20800

It’s linear so you map out the first differences and then put it into y=Mx+b form. For the you gotta find the y intercept

Answers

[tex]▪▪▪▪▪▪▪▪▪▪▪▪▪  {\huge\mathfrak{Answer}}▪▪▪▪▪▪▪▪▪▪▪▪▪▪[/tex]

Let's find the slope (m) :

[tex] \dfrac{y2 - y1}{x2 - x1} [/tex]

[tex] \dfrac{16 - 11}{9 - 6} [/tex]

[tex] \dfrac{5}{3} [/tex]

slope of given line is 5/3

now, let's plug the value of coordinates of a point lying on the line ( for example : (6 , 11) and slope (m) to find the y - intercept (c) :

[tex]y = mx + c[/tex]

[tex]11 = \bigg(\dfrac{5}{ 3} \times 6 \bigg) + c[/tex]

[tex]11 = (5 \times 2) + c[/tex]

[tex]11 = 10 + c[/tex]

[tex]c = 11 - 10[/tex]

[tex]c = 1[/tex]

value of y - intercept (c) = 1, so our equation will be :

[tex]y = mx + c[/tex]

[tex]y = \dfrac{5}{3}x + 1[/tex]

In the figure BD||EG and the measure of angleACB=35 degree. What is the measure of angle GFH
A 35. B 55. C 135. D 145

Answers

Answer:

35 degree

Step-by-step explanation:

same degree as GFH

A girl has 7 coins in her purse. She has six 10c coins and one 50c coin.
She takes two coins at random from her purse,one after the other.

a)​Draw a tree diagram to show all the possible outcomes.

Answers

Either 4 10c 1 50c coin or 5 10c since

if she has 6 10c coins and 1 50c coins she threw 2 away only options would you take 10c coin and 1 50c or 2 10 c hope this helps

a pack of four chocolate balls cost £1.20.what is the price per unit.

Answers

The cost of 4 chocolates bars is £1.20. The cost of 1 chocolate bars is . The cost of 1 chocolate bars is £0.3. Therefore, the price per unit of chocolate bars is £0.3.

Elsa works as a tutor for $15 an hour and as a waitress for $9 an hour. This month, she worked a combined total of 101 hours at her two jobs. Let tbe the number of hours Elsa worked as a tutor this month. Write an expression for the combined total dollar amount she earned this month.

Answers

Answer:

6t + 909

Step-by-step explanation:

t - tutor (hours)

101 - t = waitress (hours)

$15t + $9(101-t) = 15t + 909 - 9t

6t + 909

What are the roots (x intercepts ) of the equations below ? Show your work , Factor if possible or use Quadratic Equation . a ) y = x ^ 2 - x - 6 b). y = 2x ^ 2 + x - 6

Answers

Answer:

b.) y = 2x ^ 2 + x - 6

Step-by-step explanation:

carry on learning

quick, please help (very easy)

Answers

Answer:

5+4c=25

Step-by-step explanation:

Answer:

5 + 4c = 25

Step-by-step explanation:

The 5 dollars spent on paper is separate from the price of all the candy, so you will add that to the total of the candy.You bought 4 pieces of candy, you that value can be changed.You spent a total of 25 dollars.We have all the info to set up an equation:

5 + 4c = 25

p.s: i have a piece of candy?

find the slope
(8, 10), (-7, 14)

Answers

Answer:

4/-15 is the slope

Step-by-step explanation:

Cross products: what number is missing?

Answers

Answer:

hi

Step-by-step explanation:

hope it helps have a nice day

Lincoln needs to order some new supplies for the restaurant where he works. The
restaurant needs at least 355 knives. There are currently 317 knives. If each set on
sale contains 20 knives, which inequality can be used to determine s, the minimum
number of sets of knives Lincoln should buy?

Answers

Answer:

x>355

Step-by-step explanation:

Answer:

x>355

...

...

...

...

...

1. The list shows the amounts of money student council collected from students for a field trip. Each student paid the same amount. What is the most the field trip could cost per student? Wednesday $36 Thursday $54 Friday $72 The most the field trip could cost is $ per student.​

Answers

Answer:

Step-by-step explanation:

First find the LCM and it is going to be 18.

what is 9 1/2 + (-4.0) .Please show work. (ps. it can be in fraction or decimal form)

Answers

Answer:

5 1/2

Step-by-step explanation:

Answer:

5 1/2

Step-by-step explanation:

so since nine is the whole number part of "9 1/2" you can easily subtract 4 from if (bc it's another whole number) but the confusing part of this problem is the "+(-4)" the parentheses are just so you don't mix up the + and -, but regardless, when you add a negative number it's the same as subtracting.

9 1/2

-4

-----------

-5 1/2

* bring down the 1/2 :)

a family sold a home for $425 000 and paid 6% commission.
a) how much was the commission?
b) how much did the family receive for the house?
c) what if the family had been able to pay only 5% commission. How much would the family have saved?

Answers

Answer:

Step-by-step explanation:

To do this take sale price, multiply it by the commission percentage, divide it by 100

sketch the graph of the equation 3y+4x=2

Answers

Simplifying

3y + 4x = 2

Reorder the terms:

4x + 3y = 2

Solving

4x + 3y = 2

Solving for variable 'x'.

Move all terms containing x to the left, all other terms to the right.

Add '-3y' to each side of the equation.

4x + 3y + -3y = 2 + -3y

Combine like terms: 3y + -3y = 0

4x + 0 = 2 + -3y

4x = 2 + -3y

Divide each side by '4'.

x = 0.5 + -0.75y

Simplifying

x = 0.5 + -0.75y

The line connecting two coordinates (0,66)  and (0.5,0) show below.

What is Mathematical equation ?

By definition, an equation is a mathematical statement made up of two expressions linked by an equal sign. In other words the equation represents the graph between x and y that is for each value of x there exist one value of y.

Here, the given equation is :

3y + 4x = 2

now, to sketch the equation in graph we need to find the two coordinate first,

Let, x= 0 then y will be :

3y+4x=2

3y = 2

y = 2/3 = 0.66

that is first coordinate is (0,66)

Now, for y = 0 , x will be :

3y+4x=2

4x = 2

x = 0.5

that is second coordinate will be (0.5 , 0)

Now, draw the line connecting these two points show below.

check and know more about equation of line here :

https://brainly.com/question/21511618

#SPJ2

What is the slope of the line passing through the points (−3,−5) and (−1,−6)?

−43

−12

12

34

Answers

Answer:

-1/2

Step-by-step explanation:

1. The formula to find the slope given two points is [tex]\frac{y_2-y_1}{x_2-x_1}[/tex]. Now, all we have to do is plug in the values.

2. (Solving)

[tex]\frac{(-6)-(-5)}{(-1)-(-3)}[/tex]  [tex]\frac{-6+5}{-1+3}[/tex] [tex]\frac{-1}{2}[/tex] [tex]-\frac{1}{2}[/tex]

Therefore, the slope is -1/2.

Answer:

B. - 1/2

Step-by-step explanation:

Use slope formula and substitute given coordinates:

[tex]m=(y_2-y_1)/(x_2-x_1)[/tex][tex]m =(-6-(-5))/(-1-(-3))=-1/2[/tex]

The inverse of f(x) is a function

Answers

Answer:

A) True

Step-by-step explanation:

The inverse of any function is itself a function.

The answer is BBBBBBBBBBBBB

An item is marked down to $357. If the original price is $525, what percent of the original price does the customer have to pay?

Answers

Answer:

68%

Step-by-step explanation:

hope this helps uwu

Item 2 Simplify the linear expression. −53a+18−16a−12 Enter your answer as simplified fractions in the boxes.

Answers

The simplified value of the expression  - 53a + 18 - 16a - 12 is  - 69a + 6.

What is a linear equation?

A linear equation is an algebraic equation of degree one. In general, the variable or the variables(in the case of a linear equation in two variables) the variables are x and y.

Given, a linear expression - 53a + 18 - 16a - 12.

First, we'll arrange the like terms

= - 53a - 16a + 18 - 12.

= - 69a + 6.

= - 3(23a - 2).

learn more about linear expression here :

https://brainly.com/question/819568

#SPJ2

 

11. James arrived at work at 8:15 A.M. and left work at 10:30 P.M. If James gets paid by the hour at a rate
of $10 and time and 1/2 for any hours worked over 8 in a day. How much did James get paid?
$140.50
$173.75
$180.75
O $187.25

Answers

The answer is 18075 because of what the problem is stating

Answer:

173. i could be wrong but if i understood the question correctly

Step-by-step explanation:

Other Questions
( 6 100 6 6 ) 3 9 3 (0)Quinn keeps a weekly blog about the horseback riding lessons she takes. One of her entries read(1) I'm no seasoned veteran, but I would at the very least invest in a sturdy pair of boots. (2) Thefriend Brian suggests buying riding breeches, which fit snugly and won't chafe, (4) The jeans I'veWhy does Ouinn include sentence 4 in her blog entryA to contrast her opinion with Briarts by saying jeans work fine for ridingBto advise her readers on which kind of jeans work best for horseback ridingC acknowledge that she's not an expert rider, but she has had some experience A witch travels 240 miles in 6 hours on her broom. Atthis rate, how far could the witch travel in 36 hours? What caused the Acadian population to migrate to Louisiana? A. They were fleeing the Haitian Revolution. B. They wanted a new life in America. C. They wanted to be near the Mississippi. D. They were expelled from their homes in Canada. Directions: Match the word with its definition. Insert the letter of the definition on the line in front of the numbered word it defines.1. conditionally A. state of being protective2. abusively B. lack of pretence3. naturalness C. with extreme offense and insult4. horizontally D. parallel to the horizon5. defensiveness E. with one or more reservation WORDS WITH MULTIPLE SUFFIXES Directions: Select the correct word to complete each sentence.1.The play was good but it was (amateurishness/amateurishly) performed.2.Thoms (carefulness/carelessness) led him to drop all the plates on the floor.3.Elsie was (understandably/understandability) shaken when she was nearly hit by a car. MARKING BRAINLIEST !! please help asap !Describe the end behavior of the graph. Then determine whether it represents me odd-degree or an even-degree polynomial function and state the number of real zeros. ______ is the rapid growth of a metropolitan area characterized by single-use zoning. What was the result of John overdrawing his checking account?Johns payment was greater than the balance he had available in his account so the bank will deposit $237.81 into Johns account so he can pay his bills.Johns payment was greater than the balance he had available in his account so he was charged a $35 fee.Johns payment was greater than the balance he had available in his account so he was charged a $5 monthly maintenance fee. Johns payment was greater than the balance he had available in his account so the bank will transfer $35 from a linked account. Antonne can build 4 towers using 68 blocks. What equation can be used to model the total number of blocks y needed for the number of towers x? Write the equation in the form y=kx . Please Heeelp Why was the Tea Act of 1773 passed? Y=a|x-h|+k11. how does the value of k affect the graph?12. how does the value of h affect the graph?13. how do the sign and absolute value of A affect the graph? Which of the following events sparked the French and Indian war If y varies directly as x and inversely as Z, and y = 22 when x=4 and z= 6, find y when x= 10 and z=25. I give Brainliest! The Grade Six pupils conducted an experiment on mixtures. One of the activities conducted was to mix sugar with water. What kind of mixture is formed? A.SuspensionB.ColloidC.SolutionD.Compound A lumberyard wanted to do a survey to see if its customers were interested in purchasingrope along with lumber. Four different groups volunteered to do the study for them. Whichgroup should do the study so the bias is minimal? One side of a triangle is X another is X+3 and the third is 2X-1. If the perimeter is 66 inches what is the length of each piece If you answer the whole question and show your work/coding I will rate 5 stars/brainliest!!! Walnut Orchard has two farms that grow wheat and corn. Because of different soil conditions, there are differences in the yields and costs of growing crops on the two farms. The yields and costs are shown in the following table. Each farm has 100 acres available for cultivation. 11,000 bushels of wheat and 7,000 bushels of corn must be grown. Please have an LP model to minimize the total cost while meeting the demand and solve it with Lindo or Excel. You need to have all parts of a model: notation, objective function, constraints, and sign restrictions. turn overturn outget through What I really love to do________________with my family is ______________________________________________with my male friends is _________________________________________________________with my female friends is _______________________________________________________ For GJ in triangle GHJ, what is the corresponding segment in similar triangle HIJ?